« first day (35 days earlier)      last day (539 days later) » 
00:00 - 15:0015:00 - 00:00

12:00 AM
0
Q: Is it obvious that this integral converges given the following assumptions?

Sherif F.The integral is [math]\int_{p(x) > 0}p^{-\lambda + 1}(x) \left| \ln p(x) \right|^k dx[/math]. Assumptions: [math]\lambda > 0[/math] [math]\int_{p(x) > 0}p^{-\lambda + 1}(x) dx < \infty[/math] [math]k > 0[/math] [math]p[/math] is a probability distribution This comes from the first few parag...

This site uses MathJax formatting of formulas. More tips here. (autocomment)Normal Human 20 secs ago
 
12:11 AM
0
Q: Finding all induced subgrapshs in a given graph

Mourya KoluguriIs it possible to find all the induced subgraphs in a given graph? If so, how?

0
Q: When is a product of two matrices in tensor product form?

TurboGiven $A,B\in\Bbb K^{n\times n}$ where $\Bbb K$ is a field when is there a matrix $R$ such that $$B^{-1}A=R\otimes R$$ $$RR'=I$$ where $'$ refers to transpose?

 
0
Q: Should we ban the author if there's spam/rude flags on his posts?

Kevin GuanFrom What are the spam and offensive flags, and how do they work?: 3 flags on a question (spam or offensive): question is banished from the front page and all question lists except search results. 6 flags (spam or offensive): post is locked, deleted, and the first revision owner loses 100 reput...

0
Q: Regarding missing points earned on stackoverflow

UnnikrishnanWhen I had earned over 200 points, the next day all of a sudden, I see my points down to 127. What is the reason for this.

 
0
Q: Research in classical algebraic topology

user238194Are there any recent/modern developments in research in classical algebraic topology? Are there still any major open problems?

0
Q: Cardinality of set difference of finite sets

moatPylonIs $|A \setminus B| = |A| - |A \cap B|$, where $A$ and $B$ are finite sets, true? I have been unable to prove this or find a good reference on cardinality of set differences. The only reference I found was ProofWiki, and the only case they consider is when $A \subseteq B$, which is not necessaril...

0
Q: SAT and NAE-SAT

DesperateCompSciStudentDescribe a polynomial-time transformation TRAN that takes an instance of SAT and transforms it into an instance of NAE-SAT (the problem where, given a Boolean expression in CNF form, you are asked, does there exist a solution that makes it true, and also makes at least one literal in each clause ...

Welcome to Math.SE, DesperateCompSciStudent. Consider adding a tag for a broader subject area to which the question belongs. Some of these tags might fit. (autocomment)Normal Human 21 secs ago
0
Q: Question about open sets in the product topology in R^N

KoriHi guys I have a statement that may be true or false. I believe it is false but I cannot prove or even show by example my claim. If $U$ is open in $R^N$ then $U= \cup _{n=1} ^\infty U_n$ where each $U_n$ is open in R. I think this is false because I know that even for $X \times Y$ not every ope...

Words such as question do not add information to titles. Please edit the title so that it better describes the specifics of your question. Do not hesitate to make it longer or include a formula if needed. More tips here. (from a bot)Normal Human 21 secs ago
0
Q: $\frac{1}{n} \sum_{i=1}^{n} 1_{A_i}-\frac{1}{n}\sum_{i=1}^{n}P(A_i) \rightarrow^{P} 0$

Rob GreenIf $\{A_n,n\geq 1\}$ are independent events show that $$\frac{1}{n} \sum_{i=1}^{n} 1_{A_i}-\frac{1}{n}\sum_{i=1}^{n}P(A_i) \rightarrow^{P} 0$$. Proof so far $P(|\frac{1}{n} \sum_{i=1}^{n} 1_{A_i}-\frac{1}{n}\sum_{i=1}^{n}P(A_i)|\geq \epsilon)=P(|\frac{1}{n} \big( \sum_{i=1}^{n} 1_{A_i}-\sum_{i=...

Welcome to Math.SE, Rob Green. A title should not be all-MathJax; having some plain text helps with search and navigation. (from a bot)Normal Human 21 secs ago
0
Q: Large Number Modulo Computation

tvasileCompute the following: a) 2^208 (π‘šπ‘œπ‘‘ 53) b) 2^288 (π‘šπ‘œπ‘‘ 73) c) 7^19 (π‘šπ‘œπ‘‘ 28) Here is what I did so far: a) βˆ… (53) = 52 (2^52)^4 => (1)^4 => 1 b) βˆ… (73) = 72 (2^72)^4 => (1)^4 => 1 c) βˆ… (28) = 12 7^12 * 7^7 mod 28 1 * 7^7 mod 28 => 7 Just wondering if what I've worked out is co...

This site uses MathJax formatting of formulas. Consider adding a tag for a broader subject area to which the question belongs. Some of these tags might fit. More tips here. (autocomment)Normal Human 21 secs ago
0
Q: Polynomial rings and degrees

NidGive an example, if possible, of two polynomials $f(x)$ and $g(x)$ in the indicated rings such that the degree of $f(x)Β· g(x)$ is not equal to the sum of the degrees of $f(x)$ and $g(x)$. If not possible, explain why not. a) $Z_8[x]$ b) $Z_7[x]$ I am very confused as to how to begin this prob...

0
Q: Percentages , cross multiply percentage gain.

derrickif $10 is 0%, what percentage gain would $30 be. Having trouble coming up with the way to visualize this in cross multiplication

0
Q: Find the upper and lower sums for $f\left(x\right)=\begin{cases} 1 & 1\leq x \leq 2\\ 2 & 2<x\leq 3\\ 3 & 3<x\leq 4 \end{cases} $ on [1,4]

joshFind the upper and lower sums for $f\left(x\right)=\begin{cases} 1 & 1\leq x \leq 2\\ 2 & 2<x\leq 3\\ 3 & 3<x\leq 4 \end{cases} $ on [1,4] Attempt $$I_{1} = [1,2-a] \ with \ sup(f) = 1 \ and \ inf(f) = 1$$ $$I_{2} = [2-a,3+a] \ with \ sup(f) = 2 \ and \ inf(f) = 2$$ $$I_{3} = [3+a,4] \ with\ su...

Title contains \begin. [Find the upper and lower sums for $f\left(x\right)=\begin{cases} 1 & 1\leq x \leq 2\\ 2 & 2<x\leq 3\\ 3 & 3<x\leq 4 \end{cases} $ on [1,4]](math.stackexchange.com/q/1563424)
0
Q: How to do complete the square on a simple quadratic form?

White House Fence JumperSuppose I have: $B = \mathbb{R}^{n \times n}, w, z \in \mathbb{R}^n$ $f(w,z) = w^TBw - 2z^TBw + z^TBz$ Suppose $B$ is nice, invertible, symmetric, whatever helps in doing the completing the square. How can we manipulate this step by step as in the scalar case? to get $j^TBj$, where $j$ is some...

0
Q: Relative position of two lines in space defined by two equations

DaetoI have two lines in space. Each defined by two equations: line a: x + y -z = -1 x + y + z = 1 line b: 2x + 2y -2z = 3 x + 2y -z = 1 How would I go about finding their relative position preferably using matrices? Thanks a lot!

This site uses MathJax formatting of formulas. More tips here. (from a bot)Normal Human 20 secs ago
0
Q: Help with a pre-calculus worksheet?

user296537Does anyone understand how to do this? I'm not great with math at all, and I'm just lost. http://imgur.com/WiiLB5p

 
1:10 AM
0
Q: Elementary graph questions (periodicity and strongly connected components)

DrWatsonJust going through some graph theory concepts and I have two elementary questions which must be pretty trivial (but sometimes what seems trivial to me turns out to be wrong, so I'd be happy if someone could confirm that the following are indeed true): (1) A periodic graph cannot self-loops - ...

0
Q: structurally unstable for nonlinear system

MikeConstruct the local phase portrait for the system x'=-y+xy y'= x+ (x^2 -y^2 )/2 and show that it is structurally unstable.

This site uses MathJax formatting of formulas. More tips here. (from a bot)Normal Human 21 secs ago
0
Q: If f, g ∈ C[0,1] with f(x) < g(x) for all x ∈ [0,1] Then Prove that there is a polynomial p(x) so that f(x) < p(x) < g(x), x ∈ [0,1]

user145993Let f, g ∈ C[0,1] with f(x) < g(x) for all x ∈ [0,1]. (i) Prove that there is a polynomial p(x) so that f(x) < p(x) < g(x), x ∈ [0,1]. (ii) Prove that there is an increasing sequence of polynomials {pn(x)} so that f(x) < p_n(x) < g(x), x ∈ [0,1], and p_n → g uniformly on [0,1]. I think in these ...

This site uses MathJax formatting of formulas. More tips here. (autocomment)Normal Human 20 secs ago
0
Q: Write x as function of y with sin

cpageIs it possible to write the equation $x+y\sin{x}=1$ in terms of $x=f(y$)? I can easily solve for $y$, but am not sure how to approach $x$.

0
Q: Damping iterations

DoeDamping is a way of taming a nonconvergent iteration to get it to converge. Given a splitting matrix $M$, which gives the iteration $$x^{k+1} = x^{k} + M^{-1}r^{k}$$ where $$r^{k} = b-Ax^{k}$$ the corresponding damped iteration with damping factor $\omega < 1$ is defined by $$x^{k+1} = x^{k} +\...

Short title. Damping iterations
0
Q: Understanding the proof of this theorem leading up to Dedekind's theorem

Kevin ShengI am reading Murty & Esmonde's Problems in Algebraic Number Theory and was wondering if anyone can offer some clarification on the proof of this theorem: Let $p \in \mathbb{Z}$ be prime and $\mathfrak{p}$ be a prime ideal of $\mathcal{O}_K$ and $\mathcal{D}$ the different of $K$. If $\mathfra...

0
Q: Projective Geometry: Complete Quadrilateral and Complete Quadrangle

Alyssa WallaceI would like help just drawing this figure. I understand the differences between the complete quadrilateral and quadrangle but I fear I am not going to draw this correctly in order to complete the proof. Prove: If a complete quadrangle PQRS and complete quadrilateral pqrs are so situated that t...

 
0
Q: I cannot access this stackoverflow question?

Eng.FouadI am trying to access this link: How to redirect to the referrer in a filter? I got HTTP 200 response with an empty page: <html> <head> <meta name="viewport" content="width=device-width, minimum-scale=0.1"> <title>how-to-redirect-to-the-referrer-in-a-filter (1Γ—1)</title> <...

 
0
Q: Computerized search for integer identities. Suggestion of representation

Enjoys MathI'm writing an app in Nim to search for curious integer identities such as those ones listed on:enter link description here where it says "curious identities derived using modular theory". The program doesn't prove the identity but should only give a recommendation of what identities for mathema...

0
Q: Should I use $dS$ or $dr$?

OviThe question states "Integrate $f(x, y, z) = x + \sqrt y -z^2$ over the patch $C = C_1 + C_2$ and they proceed to give $C_1$ and $C_2$ as $r_1(t)$ and $r_1(t)$. I've been dealing with 2 forms of line integrals so far, integrations with respect to $dS$ where $dS = ||r'(t)||dt$ and integrals with $...

 
1:40 AM
0
Q: The intersection of an infinite number of subspaces is a subspace

Arcane1729Let $V$ be a finite dimensional vectorspace over a field $\mathbb{ F}$. It's easy to show that if $U$ and $V$ are subspaces of $V$ then $U \cap V$ is a subspace. But what if there are an infinite number of them? I have seen sometimes in pure maths that you take an arbitrary finite collection of s...

0
Q: Series with double Sums?

slydezThe problem is: $$\sum_{n=1}^\infty nx (\prod_{k=1}^n\dfrac{sin^2(k\theta)}{1+x^2+cos^2k\theta}) $$ Is that "$\prod$" the same as "$\sum$"? I don't know how to start. What I tried doing was focusing on the $\prod$ equation. Which I concluded is convergent by comparing $$\dfrac{sin^2(k\theta)...

0
Q: Have I understood the question properly? Annuities in Actuarial math

John TrailI am wondering if I have interpreted the language correctly in the following question The force of interest at time $t$ is given by $\delta(t) = 0.05-0.005t$ for $\leq t < 5$ and $\delta(t)=0.02+0.001t$ for $5 \leq t \leq 10$. Calculate the present value of an annuity of $\$1000$ payable annu...

0
Q: How to show that $\int_\Omega u dv =\int _{\partial\Omega} uv dx-\int_\Omega vdu$?

lanse7ptyIt should be a primary question,but only indefinite integral version I can find in wiki.

0
Q: Find Area Under a Graph

user3002680Let f be a function defined (piecewise) on the interval [1, 2015) by the formula: f(x)= (2/k(k + 2)) , for x ∈ [k,k+1), where k=1,...,2014. Find the area of the region bounded by the graph of f, the x-axis and the lines x = 1 and x = 2015.

0
Q: Finding analyticity

user121955Given that f is analytic, under what conditions is $g(z)=\overline{f(z)}$ analytic? Does this explanation make sense? : $g'(z)=lim_{h\rightarrow 0} \dfrac{g(z+h)-g(z)}{h}=lim_{h\rightarrow 0}\dfrac{\overline{f(z+h)}-\overline{f(z)}}{h}=\overline{f'(z)}$ Therefore, g(z) is analytic when $g'(z)=\...

Short title. Finding analyticity
0
Q: Consider the given set, explain the following properties?

H.AgarwalConsider the set A = {a+b ● √5 | a,b ∈ | R, |R - the set of real numbers } Let be defined as follows: (a + b ● √5) βŠ• (c + d ● √5) = (a + c) + (b + d) ● √5 (a + b ● √5) * (c + d ● √5) = (a ● c + 5 ● b ● d) + (a ● d + b ● c) ● √5 c ΞΈ (a + b ● √5) = c ● a + c ● b ● √5 i) Is a group? Explain ii)...

This site uses MathJax formatting of formulas. More tips here. (from a bot)Normal Human 21 secs ago
0
Q: Graph Theory Problem (Algorithms)

Mutating Algorithm I'm having difficulty in approaching this problem and coming up with a solution. I'm not sure where to start in answering this question.

Title contains problem. Short question. Graph Theory Problem (Algorithms)
0
Q: Units in Polynomial Rings

NidGive an example of a natural number $n > 1$ and a polynomial $f(x) ∈ \Bbb Z_n[x]$ of degree $> 0$ that is a unit in $\Bbb Z_n[x]$. I am trying to understand how units work in polynomial rings. My book doesn't really define it and I need a bit of help with this.

0
Q: $GL(2,\mathbb{Z})$ and nilpotency

mathl0v3rI am currently reviewing some examples of nilpotent groups. Why isn't $GL(2,\mathbb{Z})$ nilpotent?

Short title. Short question. $GL(2,\mathbb{Z})$ and nilpotency
 
2:20 AM
0
Q: Simplying a recurrence

Aruka JDefine: $$R_k(n) = R_k(n-1) - R_k(n-k-1)/n$$ With $R_k(n) = 1$ for $n \leq k$. Is there a closed-form in terms of combinatorics or harmonic numbers?

Short title. Short question. Simplying a recurrence
0
Q: Some basic technical questions on the Leibniz integral rule (differentiation under the integral),

Lebron JamesOn the first paragraph of the Wikipedia page regarding the Leibniz integral rule, we get the expression $$\frac{d}{dx} \int_{y_0}^{y_1} f(x,y)dy = \int_{y_0}^{y_1} f_x(x,y)dy $$ and it says that "provided that both $f$ and $f_x$ are continuous over some region $[x_0,x_1]X[y_0,y_1]$ Can we r...

0
Q: Team Fortress 2 Optimization

Terrence TownIn Team Fortress $\mathbf{I}\mathbb{I}$ (Team Fortress $2$), there are 9 classes, and soldier is the best. Each of the 9 classes has (for simplicity) 5 types of weapons for each of 3 weapon slots. Additionally, there are up to 12 players on each team, and there are 2 teams. Also, Valve allows you...

0
Q: Find the point on the plane x + 2y + 3z = 4 that is closest to the point (1, 1, 2) using Lagrange Mulipliers

BNSlugI know how to solve this problem "normally" (using vector methods) but I don't know how to solve it using Lagrange multipliers.

0
Q: Family of all elementary subsets of $\mathbb{R}^n$ is a ring, but not a $\sigma$-ring

RiverI was going through the introduction to Lebesgue theory in Baby Rudin, where the property was given that: $\mathcal{E}$ is a ring, but not a $\sigma$-ring. $\mathcal{E}$ here represents the family of all elementary subsets of $\mathbb{R}^n$, $n$-dimensional Euclidean space. An elementary s...

 
0
Q: How do I flag an improper edit of a title?

aparente001In a question (Can I write a paper on a method that is worse than existing ones?), someone edited the title and introduced the word "wrong" which I believe was entirely unwarranted by the question. How do I flag this? From the discussion I have had with the person who made the edit, I anticipat...

 
2:41 AM
0
Q: Integrating using change of variables

user296558Change to uv variables, find the Jacobian and evaluate ∬e^(-(xy)/2)dA, use transformation x=sqrt(v/u) and y=sqrt(uv). R is a region in the first quadrant bounded by y=x/4, y=2x, y=1/x, and y=4/x. I found the Jacobian, which is -1/2u. u ranges from 1/4 to 4, and v from 1 to 2. Doing the actual in...

Welcome to Math.SE, user296558. This site uses MathJax formatting of formulas. More tips here. (autocomment)Normal Human 21 secs ago
0
Q: Integration General Technique

RSparkesLet $P(x) = a_nx^n + ..... a_1x + a_0$ Is there a general technique for integrating $$\frac{P(x)}{\sqrt{Ax^2 + Bx + C}}$$ Similarly is there a general method for $$\frac{1}{P(x)\sqrt{Ax^2 + Bx + C}}$$ And finally is there a general method for $$\frac{P(x)}{Q(x)\sqrt{Ax^2 + Bx + C}}$$ Wh...

 
3:03 AM
0
Q: Oriented simple graphs

56ryasqcnI've been stuck on this for a while: The number of simple digraphs with |V| = 3 and exactly 3 edges is (a) 92 (b) 88 (c) 80 (d) 84 (e) 76 The answer is (d) though i don't know how to arrive there.

0
Q: How to write IF… as mathematics equation (Linear function)

user296561i would like to write if function as mathematics function By if {y(i)=1,0,1}; How i can write it.

0
Q: Finding Positive and Negative Limit Sets (Ο‰(Ξ“) and Ξ±(Ξ“))

yung_PabsSo this problem statement says "Use analytical or graphical techniques to find the positive and the negative limit sets of the orbits through the listed initial points." I have a system $$x^{'}=y$$ $$y^{'}=x$$ and the points $(1,1), (1,-1), (1,0)$ I have read through my notes and book and am st...

0
Q: Prove by contradiction that for a transitive relation β„› on A, β„›2 is also transitive

RiGidProve by contradiction that for a transitive relation β„› on A, β„›2 is also transitive. In order for a relation to be transitive it must $aRc$ ^ $bRc -> aRc$

0
Q: Collatz 4n+1 rule?

Philip TsangI noticed something about the Collatz Conjecture, (I was literally obsessed with trying to prove it). I of course have NO intention of trying to prove it, clearly it is beyond my reach and I hope not to offend anyone by what may be a nonsensical observation, but I was a bit curious. This is wha...

Questions tend to get more attention when they have a tag for a broad area of mathematics relevant to the question. Some of these tags might fit. (from a bot)Normal Human 21 secs ago
0
Q: Correlation Coefficient Var(X)

user270494I'm having trouble understanding how to find Var(X) and Var(Y) for the correlation coefficient between Z and W. Z = 2X + Y W = X - 3Y I have Cov(Z, W) = Cov(2X + Y , X - 3Y ) = Cov(2X, X) + Cov(2X, -3Y ) + Cov(Y , X) + Cov(Y , -3Y ) = 2Cov(X, X) - 6Cov(X, Y ) + Cov(Y , X) - 3Cov(Y , Y ) ...

This site uses MathJax formatting of formulas. More tips here. (from a bot)Normal Human 21 secs ago
0
Q: Follow up question to what is the $2006^{\text{th}}$ term in the sequence?

user19405892In a sequence of integers, $t_1, t_2, \ldots; t_{n+3} = t_n+t_{n+1}-t_{n+2}$ for $n \geq 1$. If the first three terms, in order, are $1$, $3$, and $6$, what is the $2006^{\text{th}}$ term? There was an answer to my question which said that $t_{2n+1} = 1+5n$ and $t_{2n} = 8-5n$. I want to prove t...

0
Q: intgerate ${\displaystyle \intop_0^2 f\left(x\right)\,dx}$ using the $U\left(f,P\right),L\left(f,P\right)$

mikelet $f$ be a step function, $ f:\left[0,2\right]\longrightarrow\mathbb{R} , f\left(x\right)=\begin{cases} 1 & 0\leq x<1\\ 3 & x=1\\ 2 & 1<x\leq2 \end{cases} $ intgerate ${\displaystyle \intop_0^2 f\left(x\right)\,dx}$ using the $U\left(f,P\right),L\left(f,P\right)$ definition. attempt Let...

0
Q: What trigonometric functions are constructable?

CsunWG _Given a trigonometric function, can you tell if it is Constructible? If I have $\sin(n)$ for example, how would you tell if it is of the constructible set of numbers?

Tall formulas in titles break the layout of question lists. Please remove \displaystyle in the title. (autocomment)Normal Human 28 secs ago
0
Q: Discrete Recurrence Relation

Guerrero DarleneFind a recurrence relation that counts the number off-diagonal elements of an n + 1 x n + 1 matrix. Solve this recurrence relation for an expression of the number of off diagonal entries as a function of n.

0
Q: What property does this binomial probability calulation used?

PatrickI have an equation $$\sum_{k=2}^7{7\choose k}{0.01^k}(1-0.01)^{7-k} = 1-(0.99)^7 - 0.07(0.01)(0.99)^6 \approx 0.002031$$ I don't know what property the teacher used to quickly transform the summation to two simple equations. Can someone please give me a hint? P.S. This formula is used to calcu...

0
Q: Find the volume of the solid bounded by the surfaces x = 1 βˆ’ y^2 , x = βˆ’1 and z^2 = 1 βˆ’ x.

GuestCalcI am Having a bit of difficulty setting up the bounds for this question. So far I have got: -1 <= x <= 1-y^2 -sqrt(2) <= y <= sqrt(2) -sqrt(2) <= z <= sqrt(2) Can someone please confirm if I am on the right track?

Welcome to Math.SE, GuestCalc. This site uses MathJax formatting of formulas. More tips here. (autocomment)Normal Human 21 secs ago
0
Q: Smoothing transverse self-intersection

ceiLet $S$ be a complex surface, and let $C \subset S$ be an immersed complex curve with a transverse self-intersection at point $P$. Let $\tilde{C}$ be a curve obtained from $C$ by smoothing the intersection at $P$. Is there a formula for computing the genus $g(\tilde{C})$ in terms of $g(C)$? -- A ...

0
Q: The proper of open and coutinuous mapping

Nathapon Kin KinI'm proving but get many problems $f$ is continuous and open mapping if and only if $\overline{f^{-1}(B)}=f^{-1}(\overline{B})$

0
Q: we have to prove that f on [a,b] have only finite number of zeros.

EklavyaLet $f:[a,b]\to\mathbb{R}$ be differentiable.Assume that there exists no $x\in[a,b]$ such that $f(x)=0=f'(x).$prove that the set {$t\in[a,b]:f(t)=0$} of zeros of $f$ is finite.

Short question. [we have to prove that f on [a,b] have only finite number of zeros.](math.stackexchange.com/q/1563627)
0
Q: Divisible by positive n integers using Induction

Guerrero DarleneProve by induction that 3n +7n - 2 is divisible by 8 for all positive integers n.

 
3:55 AM
1
Q: Images in firefox?

Ruchir BaroniaI use both Google Chrome and Firefox, but the Stack Overflow website works differently on both websites. This is for images. When directed to this screen: In google chrome, we can just use ctrl + v to paste the image after using something like snipping tool or print screen. In Firefox howeve...

 
0
Q: Sequence Converge or Diverge

User123I am trying to figure out if this sequence will either diverge or converge. My initial thought was that any value of n to sin will be less than 1. enter image description here

Short title. Short question. Sequence Converge or Diverge
0
Q: Context free grammar L={a^m b^n c^k| n,m,k >= 0, n>= m+k}

ImTheRealOneWrite a context free grammar for the following: L={a^{m} b^{n} c^{k}| n,m,k >= 0, n>= m+k} I am having some trouble writing the above cfg from the language. So far, I have S-> S1 S2 S3 S1 -> a S1 S2 S1 -> \Lambda S2 -> b S2 S2 -> \Lambda S3 -> S2 c S3 S3 -> \Lambda

 
0
Q: HOW-TO questions are treated in different ways

Danny ChenI can see a lot of HOW-TO questions when reviewing, most of which are asked by new users, and probably will be closed. Some of them are closed reasonably because the questions are simply "why not working" without even a code snippet. However a few of them are actually asked properly, the user e...

 
4:12 AM
0
Q: Find limit approaching 2 of $(x^2-4)/sin(x-2)$ without using l'hopitals rule

Ilia LabkovskyThis is an indeterminate form and I think I should use the fact that x-2/sin(x-2) = 1 but idk how to do that.

0
Q: This one is quite challenging! wasted my time on this yet did not get the answer.

Hrishikesh Murali\lim_{a \rightarrow \infty}\frac{1}{a}\int_{0}^{\infty}\frac{x^2+ax+1}{1+x^4}.tan^{-1}(\frac{1}{x})dx=?

Welcome to Math.SE, Hrishikesh Murali. This site uses MathJax formatting of formulas. More tips here. (from a bot)Normal Human 20 secs ago
0
Q: Graph Algorithms Question

Mutating Algorithm I'm trying to design the algorithm and answer these two questions but can't produce a solution. I've tried drawing the pictures for the graph as well.

Words such as question do not add information to titles. Please edit the title so that it better describes the specifics of your question. Do not hesitate to make it longer or include a formula if needed. More tips here. (from a bot)Normal Human 20 secs ago
0
Q: Is the indefinite integral of a function meaningless on its own?

ArjoonI mean to say does the function F(x), the antiderivative of f (x) have any sort of meaning if you're not taking the difference of it between two limits a and b.

0
Q: Simplying a recurrence

Aruka JDefine: $$R_k(n)=R_k(nβˆ’1)βˆ’R_k(nβˆ’kβˆ’1)/n$$ With $R_k(n)=1$ for $n≀k$. I am trying to simplify this to a closed form (binomial coefficients, harmonic numbers, etc). Is it possible?

0
Q: Clarification regarding Spivak, exercise 1-24

YouKnowNothingThe 24th problem in the first chapter of Spivak's Calculus has to do with proving that the placement of parentheses in a sum is irrelevant. Let $s(a_1, a_2,... a_{n})$ denote some sum formed from $a_1, a_2,...a_n$. For example, if $n=5$, $s(a_1, a_2,... a_{5})$ may represent $(a_1 + a_2) + (a_3 ...

0
Q: Is this true that $Tr(ABC)=Tr(ACB)$?

SmartLet $A,B,C\in M_n$. Is this true that $Tr(ABC)=Tr(ACB)$?

0
Q: Proving Lagrange's Group Theorem

RobI am trying to prove Lagrange's Group Theorem that given the group G with the subgroup H then $|H|\ |\ |G|$ and if $\{g_i|i\in I\}$ is a complete list of representatives of the cosets of H, then$\ |G|=|I||H|$ where I is the index of H in G. I was wondering if this proof is correct. $$\begin{align}&

Tagged proof-verification. Proving Lagrange's Group Theorem
0
Q: Specific sequence in $l^2$

MaciejI have a problem. I am trying to find sequence $x\in l^2$, witch satisfies: $(\forall p<2) (x\not\in l^p)$. Is it possible? Why?

Short title. Short question. Specific sequence in $l^2$
0
Q: Approximations in MATLAB

Ste_TayI'm still new to MATLAB I'm trying to calculate the gamma function in MATLAB. My code is as follows: int(power(z,2.0*b)*exp(-z),z,0,inf) My output ends up being (4851*gamma(49/50))/2500. I have to input my output in order to get the approximate solution: 1.9636. Is there a way to output the ...

Short title. Tagged matlab. Approximations in MATLAB
0
Q: find limit at 0 from the left of $(e^(1/x))*cosx$ without using l'hopital's rule

Ilia LabkovskyI would separate this limit but 1/x is an indeterminate form, which is where I'm stuck.

 
4:44 AM
0
Q: How should we handle questions that include identifying information?

Anonymous MathematicianI was looking at this question about a 17-year-old Ph.D. student in astrophysics, and I was struck by the potential for identifying this student (if it's a real scenario, which I don't for a second believe, but that's another topic). The number of current 17-year-old astrophysics Ph.D. students ...

 
4:57 AM
0
Q: Is the group Z2 x Z9 cyclic?

NicoleSo I am doing a project and was given the group Z2 x Z9 and was told it was a noncyclic group. However, when finding the subgroups (1,1), (1,2), (1,4), (1,7), and (1,8) all generate the group. Am I doing it wrong or is the group actually cyclic?

 
0
Q: Should we allow people to offer to help through outside communications like Team Viewer?

masonI've come across a user that I've seen on several occasions offer to assist others via TeamViewer. He will ask them to provide their ID and password. It appears that the user is positively trying to help and not attempting anything nefarious, but I have some concerns about this: Sharing your sc...

 
0
Q: What is "rotating/shifting rows of (sparse) matrix"?

mavaviljIn this signal processing paper(SchΓΆrkhuber, Klapuri, Sontacchi) its referred to "rotating (shifting) the rows of M^{CQT} up- or downwards.". Any idea what this means?

0
Q: Conversion into Polar Co-ordinates.

MathematicsCan someone please hint me as how to solve this category of questions as I am novice in it.

Short question. Question contains please. Conversion into Polar Co-ordinates.
0
Q: calculate mean of two variables given two regression equations

SylvesterGiven these 2 regression equations how do I compute mean and find r. X=-0.4Y+6.4 Y=-0.6X+4.6 when I rearranged the equations, I solved for X and Y hence X=6, Y=1 How do I get mean and r using the the two values.

Please don't use (self-learning) tag just because you were self-studying. This tag is only for questions about the process of self-studying. (from a bot)Normal Human 21 secs ago
0
Q: If A and B are finite sets, show |AβˆͺB|= |A|+|B| iff A∩B = βˆ…

strugglingmathlearnerI know in general that AUB = A + B - (A∩B) but then how to prove it in general using cardinality and sets?

0
Q: What is the limit approaching infinity from the negative side of $x+Sqrt(x^2+2x+3)$

Ilia LabkovskyThis is easy for normal infinity, but being negative makes taking the root impossible.

0
Q: Probability add or multiply

user256670If we have 6 balls, 2 yellow, 3 red and 1 green. What are the probabilities of drawing 2 yellow balls in 2 goes? My answer is 1st go is 1/6. Because of no replacement, 2nd go is 1/5. Now do I add the two or multiply the two. Add gives 11/30, and multiply gives 1/30

0
Q: How to find interval of convergence of series?

Nerdy Each term of $x^m$ is also a term of $x^{ln(m)}$, so interval of convergence must be smaller than $(-1,1)$. So option (a) is correct since $e>1$, but how to explicitly arrive at interval $(0,1/e)$. please suggest.

0
Q: Evaluate the sum with harmonic numbers

billybobEvaluate the following sum $\space\space\sum\limits_{n=1}^{\infty}\left(\frac{1}{n}\sum\limits_{m=1}^{n}\frac{1}{m}\right)^2$.

 
5:39 AM
0
Q: Let $S:= \{1-\cfrac{1}{2}n: n\geq0\}βŠ‚[0,1],$ and let $T:= \{m+a_n:mβˆˆβ„•, an∈S\}βŠ‚[0,\infty)$. Show that $T$ is countable.

kankuLet $S:= \{1-\cfrac{1}{2}n: n\geq0\}βŠ‚[0,1],$ and let $T:= \{m+a_n:mβˆˆβ„•, an∈S\}βŠ‚[0,\infty)$. Show that $T$ is countable. Please someone help me with this

Short question. Question contains please. [Let $S:= \{1-\cfrac{1}{2}n: n\geq0\}βŠ‚[0,1],$ and let $T:= \{m+a_n:mβˆˆβ„•, an∈S\}βŠ‚0,\infty)$. Show that $T$ is countable.
0
Q: Let $f$ be analytic function defined on the open unit disc in $\mathbb{C}$. Then which are of the following true?

user290591Let $f$ be analytic function defined on the open unit disc in $\mathbb{C}$. Then $f$ is constant if $f\left(\frac{1}{n}\right)=0$ for all $n\geq1.$ $f(z)=0$ for all $|z|=1/2$ $f\left(\frac{1}{n^2}\right)=0$ for all $n\geq1.$ $f(z)=0$ for all $z\in (-1,1)$ I used Identity theorem and conclude ...

0
Q: Convergence of (x^n)/n for x>= 0

user269711I'm a bit confused. I think that it's uniformly convergent on [0,1]. My proof: Let epsilon be greater than 0. Then, N exists in the natural numbers such that N*epsilon > 1. Then, for all x in [0,1], |fn(x) - f(x)| = x^n/n < 1/n < epsilon. But, for x in (1, inf), I'm not sure what to do. It doesn...

Title ends with a digit. Convergence of (x^n)/n for x>= 0
0
Q: Can someone explain where the map in this isometry came from?

Al JebrCan someone explain where the map in this isometry came from? First the question is: By thinking about how a circular cone can be 'unwrapped' onto the plane, write down an isometry from $$\sigma(u,v)=(u\cos v, u \sin v, u), \ \ \ \ \ u>0, \ \ 0<v<2\pi$$ (a circular half-cone with a straight lin...

Title contains someone. Tagged proof-explanation. Can someone explain where the map in this isometry came from?
 
6:30 AM
0
Q: Is infinity always a regular value of rational function from $S^1\to S^1$

user136592$f$ is a rational function $S^1\to S^1$, where $S^1$ is the one point compactification of $\mathbb R$. Is $\infty$ always a regular value of $f$?

 
0
Q: Suggestion for missing reputation on stackoverflow

Hazem AbdullahSometimes I loose some reputation, for example I get 814, the next day it become 812, and some days later it become 810, I check the Reputation page in my profile and I did not discover why I loose these points. I suppose I may edit a question and get +2 and this question is deleted now. Why th...

 
0
Q: Bounded harmonic function on whole plane

lonelyboy67Show that any bounded harmonic function on the whole plane must be a constant function.

 
0
Q: Are naming convention questions on topic?

GstestsoOK, I know there is a naming-conventions tag here, but naming often includes some degrees of opinion. Is asking about naming conventions off-topic?

 
7:04 AM
0
Q: How do I identify the indeterminate situation in this problem?

MACIdentify the type of indeterminate situation that justifies using l'Hopital's rule, then use l'Hopital's rule to find the limit. lim x->􀀏0 (e^(8/x) - 1x)^(x/2)

 
7:40 AM
0
Q: Explain this newton's method problem with critical values as your initial guess?

MACSuppose Newton's Method is used with an initial guess Xo that lies at a critical point (a, b), where b does not equal 0. What happens to X1 and later approximations? Give reasons for your answer.

 
7:57 AM
0
Q: Help find the integral

Ilia Labkovskyintegral of $(e^x-e^-x)/(e^x+e^-x)^2$

Words such as help are uninformative in titles. Please edit the title so that it better describes the specifics of your question. Do not hesitate to make it longer or include a formula if needed. More tips here. (from a bot)Normal Human 22 secs ago
0
Q: Can anyone please clarify the meaning of this statement

user71346$L:\mathbb{R}^3\to\mathbb{R}^3$ is a linear map and $S\subset\mathbb{R}^3$ is a regular surface invariant under $L$, that is $L(S)\subset S$. My first doubt is what does $L(S)\subset S$ actually mean? Does it mean the set of points of $L(S)$ is a subset of the set of points of $S$? My next que...

0
Q: Canonical class of blow-up

se0808 Let $B_1 \to X_1=\mathbb P^n$ be the blow-up of $\mathbb P^n$ along a linear subspace $\mathbb P^k$. Let $B_2 \to X_2$ be the blow up of a quadratic cone $X_2=\{x^2=yz\} \subset \mathbb A^3$ in its vertex $p$. I need to calculate $K_{B_i}$. As any blow up is an isomophism ouside of the excepti...

Words such as anyone, please do not add information to titles. Please edit the title so that it better describes the specifics of your question. Do not hesitate to make it longer or include a formula if needed. More tips here. (from a bot)Normal Human 25 secs ago
 
8:11 AM
0
Q: In graph theory, are undirected graphs assumed to be reflexive? What are the assumptions about symmetry and transitivity?

BillyThe title says it all, please help me.In graph theory, are undirected graphs assumed to be reflexive? What are the assumptions about symmetry and transitivity?

0
Q: find P(X<Y) by splitting it to interval

pasaIf I want to find P(X 1)can I write it as P(X 2) if I know X

0
Q: Help correctly setting up Stokes Theorem problem

badfilmsI am trying to evaluate the following using Stokes' Theorem, but I am running into issues setting it up... $$\iint_Scurl\vec(F)\cdot d\vec(S)$$ where $$\vec(F)(x,y,z)=\langle sin(x^2y),xy,xyz^2 \rangle$$ S is a cone $$z=\sqrt(x^2+y^2) , 0\leq z\leq 3 $$ If I parametrize it, I end up with $$9=x...

Words such as help are uninformative in titles. Please edit the title so that it better describes the specifics of your question. Do not hesitate to make it longer or include a formula if needed. More tips here. (from a bot)Normal Human 24 secs ago
0
Q: determine number of possible combinations satisfying given query

user249117I am provided with a value m , such that I have numbers from 1 to m , and another number n ( n<=m) such that I can choose any n numbers from given m numbers. Now I need to calculate total possible combinations such that the maximum of the selected n numbers is strictly less than the sum of remai...

0
Q: Solving a basic differential equation

Trent BingHow do I solve $\frac{dx}{dt} = a + (b - 1)x^2$, where $a$, $b$ are constant? I've tried pushing symbols around, but to no avail.

Title contains basic. Short question. Solving a basic differential equation
0
Q: Equivalent condition for normalizer problem

Bhaskar VashishthLet $\cal{U}$ be the unit group of group ring $\Bbb{Z}G$ then the Normalizer Problem (NP) states that $N_{\cal{U}}(G)=G\frak{z}$ where $\frak{z}=\cal{Z(U)}$. Now why (NP) is equivalent to saying that $Aut_Z(G)=Inn(G)$ where $Aut_Z(G)$ denotes the automorphisms of $G$ induced by conjugation with ...

0
Q: Finding extreme values using Lagrange Multipliers

Jermaine LawSo the question asked to find the extreme values of the function f(x,y,z)=e^(xyz) `under the constraint 2x^2+y^2+z^2=24. enter image description here I'm a bit lost on what to do now because I tried isolating for one of the variables and sub it back into another one of those 4 equations. But, i...

This site uses MathJax formatting of formulas. More tips here. (from a bot)Normal Human 20 secs ago
0
Q: Is it true that if V is a norme space then $B(V)$ is a Banach space?

Dac0I found it somewhere and copied in my notes, but couldn't find the source and it seems to me a pretty strong statement... does anybody knows if it's true and can sketch a roughly proof?

 
8:35 AM
0
Q: Integer divide by 0

vadzThis is not my error I'm just wondering what happens to SO on 5 Dec 2015 around 9 PM UTC+08:00. As I saw there's an integer divide by 0 word and wondering if it's related to why SO was offline.

 
0
Q: Verify an identity in probability

user296614Verify the identity: P(AUB)=P(A)+P(BA') I could verify using the Venn diagram. Is there another way?

0
Q: Totient euler function: Why $\gcd(n,k)=1$ is important for $\zeta _n^k$.

MSELet $\mu_n=\{\zeta \mid \zeta^n-1=0\}=\{1,\zeta _n,\zeta _n^2,...,\zeta _n^{n-1}\}$. We call generator an element $\zeta_n^k$ when $\gcd(k,n)=1$. Why those number are such important ? I think that $\left<\zeta _n^k\right>$ is a subgroup of $\left<\zeta _n\right>$ if and only if $(k,n)=1$, and tha...

Questions tend to get more attention when they have a tag for a broad area of mathematics relevant to the question. Some of these tags might fit. (from a bot)Normal Human 20 secs ago
0
Q: Big-Omega Question

P999I am currently stuck on the following question: My current thinking is to assume g $\in$ $\Omega$(f), use the definition to somehow show that ccc is c and therefore the definition of $\Omega$ holds true for $g^3$ as well. But I can't seem to put together a chain of equalities to show this. Am...

Words such as question are uninformative in titles. Please edit the title so that it better describes the specifics of your question. Do not hesitate to make it longer or include a formula if needed. More tips here. (autocomment)Normal Human 25 secs ago
0
Q: What is the proper notation for the "average of the last N elements of a sequence"?

peetonnI use sliding window average in my calculations: average for the last N elements and average for the N elements before that. What would be the correct mathematical notation to describe this?

0
Q: x in both numerator and denominator

Christopher TreadgoldI have been raking my brain for the solution to this problem for a while, and have searched as best I could for an answer. I have to solve for $x$ in the equation: $$y= \frac{x-1}{2x+3}$$ Since $(x - 1)$ is not a factor of $(2x+3)$ and I can't figure out how to seperate out x once I reach the st...

Welcome to Math.SE, Christopher Treadgold. Tag (proof-verification) should not be the only tag a question has. Please add a tag for a subject area to which the question belongs. (from a bot)Normal Human 25 secs ago
0
Q: Relation of numerical stability of matrix inversion and it's determinant

JaeJun LEEI have been taught that "inverting a square matrix with small determinant is numerically unstable because it is close to singular"? Is this right opinion?

0
Q: non-deterministic finite automaton state transition table language question

dmnteI have the nfa problem below which I dont quite understand. I understand the basics of dfa's/nfa's but have not seen the syntax used below before, and dont quite understand how to create the state transition diagram from information. If anyone could help me understand the problem below it would b...

0
Q: Diophantine equation $x^2+y^2=z^2+t^2$?

asadI would like to find a source book or article which discuss the Diophantine equation $$ x^2+y^2=z^2+t^2 $$ for which $x,z$ are odd positive and $y,t$ are even positive integers. Any brief explanation is also welcome and appreciated. Thanks!

 
9:25 AM
0
Q: Functional equation $f\left(\frac{1}{x}\right)+(x+1)f(x)=1$

billybobFind all functions $f$ such that $f\left(\frac{1}{x}\right)+(x+1)f(x)=1,\space x\neq0$.

0
Q: The Poincare-Bendixson Theorem

MikeLet f be a C^1 vector field in an open set E ΟΉ R^2 containing an annular region A with a smooth boundary. Suppose that f has no zeros in A', the closure of A, and that f is transverse to the boundary of A, pointing inward. (a) Prove that A contains a periodic orbit. (b) Prove that if A contains...

 
9:38 AM
0
Q: Help limit please

GelyaCould you tell me the way to solve this , but not the answer ? lim x^ (x^Ρ… - 1) , x-> 0 Thank you.

Words such as help, please do not add information to titles. Please edit the title so that it better describes the specifics of your question. Do not hesitate to make it longer or include a formula if needed. More tips here. (from a bot)Normal Human 23 secs ago
0
Q: is |ln|x|| differentiable?

gbox is $|ln|x||$ differentiable for all $x$ is is defined and continuous? I can see that on the graph that it is not differentiable at $-1$ and $1$, but how can I prove it?

 
10:10 AM
0
Q: Polynomial quotient ring elements with no GCD

BrunoFrom Wikipedia, a GCD domain is an integral domain in which every pair of elements has a GCD. Let us consider some polynomial quotient ring $R=K[X]/(pq)$ where $K$ is a field and $p$, $q$ are (irreducible) polynomials. Then $R$ is not an integral domain (since $pq=0$ in $R$), so cannot be a GCD ...

Questions tend to get more attention when they have a tag for a broad area of mathematics relevant to the question. Some of these tags might fit. (autocomment)Normal Human 26 secs ago
0
Q: How to solve the following double infinite series diagonally?

chetan joshiHow to solve the following double infinite series diagonally? I tried it but could not get the answer.

 
0
Q: search giving invalid results

maazzaSo I tried to search for "efficy" but only one result was valid(a typo unfortunately) and the rest didn't even contain the word, is this a bug ? the url http://stackoverflow.com/search?q=efficy edit:fixed the typo so it is no longer visible

 
10:27 AM
0
Q: Question about the definition of the adjoint representation of Lie groups

AshLet $\mathfrak{g}$ denote the Lie algebra of a Lie group $G$. The adjoint representation of $G$ is defined as the function $Ad_g:\mathfrak{g}\rightarrow\mathfrak{g}$ that maps each $x\in\mathfrak{g}$ to $Ad_g(x)=gxg^{-1}$, for some $g\in G$. why does $gxg^{-1}$ fall into $\mathfrak{g}$ ?

0
Q: Solving an equation using Matrix Calculus

zshaI have to solve the following equation for x:$$ \sum_{k} (x'A_kx - 1)(A_kx)=0$$ $A_k$ is a matrix and $x$ is a vector. Can anyone give me some suggestion on how to solve this problem?

0
Q: Let B = {0,1,2,3,4} and let {0},{1,3,4},{2} be a partition of B that induces a relation Q. Find the distinct equivalence classes of Q.

Todd BenjaminLet B = {0,1,2,3,4} and let {0},{1,3,4},{2} be a partition of B that induces a relation Q. Find the distinct equivalence classes of Q.

0
Q: Factorials in different base

user296636Got an interesting problem from a friend. How many zeroes does $n!$ end in when written in base $n$? For every factor of $n$ in $n!$, I know that there will be $1$ $0$ added. However, I'm not really sure how to proceed from here.

 
-1
Q: Why my answer has been deleted?

AnixxWhy this my answer has been deleted? http://math.stackexchange.com/a/1100879/2513 What's wrong with it?

 
0
Q: Sum about sine function II

antonio asiskeeping in mind the similar question as above I try to show that $$\sum _{n=1}^{\infty } \sum _{k=1}^{\infty } \left(\frac{4 \sin ^2\left(\frac{\pi n}{k x}\right)}{x}-\frac{2}{k x}\right)=\frac{2-x \coth \left(\frac{x}{2}\right)}{2 x}$$ but it seem a hard work...any clue thanks

0
Q: Reduce to Wallis formula

vitoHow do we get Wallis Formula $$\frac{\pi}{2}=\lim_{l\to\infty} \prod_{j=1}^{l+1}\frac{(2j)(2j)}{(2j-1)(2j-1)} $$ from $$\lim_{n\to\infty}\frac{(n+1)^2}{n+\frac{3}{2}} \bigg[\frac{\Gamma(n+1)}{\Gamma(n+\frac{3}{2})}\bigg]^2=1$$

0
Q: Exponential to polar form

Shinning EyesI have exponential form $$ je^{-j\pi/2} $$ I want to convert this to polar form $$j(cos\pi/2 + j sin \pi/2)$$ is it correct?

Short title. Short question. Exponential to polar form
0
Q: Given undergraduate Algebra background, which introductory Homological Algebra textbook?

zudumathicsI have read the answer for graduate-level Algebra background and all answers in stackexchange and mathoverflow discussing Homological Algebra textbooks. But none of them directly answers my question, and not all of the lauded textbooks indicates the prerequisites clearly. My background: undergr...

0
Q: Irreducibility of $x^4+x^3+x+1$

xndrme Show that $x^4+x^3+x+1$ is irreducible in $k[x]$ for any field $k$. I have no idea how to prove it other than writing $p(x)q(x)=x^4+x^3+x+1$ and trying to figure out what happens with the coefficients of $p$ and $q$ (supposing $p,q$ both degree 2, or $p$ degree 3 and $q$ degree 1). But I was...

0
Q: Problem with Lebesgue-integral of measurable set and Lebesgue-integrable function

monoidI am trying to show, that Let $\Omega \subset \mathbb{R}^n$ be measurable and $f\colon \Omega \rightarrow [0,\infty)$ Lebesgue-integrable. Show that: $$\int_\Omega f(x)dx=\int_0^\infty \lambda(\{f>s\})\; ds.$$ Can somebody explain to me, why for $(x,s) \in \Omega \times \mathbb{R}$ is $\chi...

0
Q: Quotient of polynomial?

MathBeginnerI'm studying abstract algebra, and I'm confused in quotient and polynomials. The exercise that I feel confused was this; Let $E$ be an extension field of a field $F$ and let $\alpha\in E$ be transcendental over $F$. Show that every element of $F(\alpha)$ that is not in $F$ is also transcendent...

0
Q: Endomorphism on $\mathbb{R^3}$

tom785I'm wondering about an endomorphism. If we have an endomorphism $f$ in $\mathbb{R^3}$ such as its matrix in canonical basis is \begin{pmatrix} 1 & 1 & 0 \\ -1 & 2 & 1 \\ 1 & 0 & 1 \\ \end{pmatrix} and $V=Vect{(1,1,1)}$, $Z=Vect((1,0,0),(0,1,-1))$ We see $V$ is invariant by $f$ and $...

0
Q: Limits using L'Hopital's rule

Luka NikolićCould you help me with this one? Thanks. The answer should be 1, somehow. I tried everything I know, but I couldn't solve it. http://i.stack.imgur.com/LuyX0.png

 
11:21 AM
0
Q: Why this question has been deleted?

AnixxI mean this question: http://math.stackexchange.com/questions/112762/is-there-a-formula-to-quickly-express-delayed-functions-in-terms-of-finite-diffe It had an accepted answer befor deletion.

 
0
Q: Does Pollard rho works for Gaussian integers?

LehsCould I expect that the Pollard rho method 1. x ← 2; y ← 2; d ← 1; 2. While d = 1: 1. x ← g(x) 2. y ← g(g(y)) 3. d ← gcd(|x - y|, n) 3. If d = n, return failure. 4. Else, return d. should works for Gaussian integers? With the minor changes due to gcd?

 
0
Q: Vote to re-open 2 year old question?

Jan DoggenI just edited this question, then noticed it was two years old (Community User put it back in a queue). I need help! PopUp with only OK button Looks like a perfectly valid question to me. It could do with some more information, which is unlikely to come now. Should I vote to reopen?

0
Q: Need a better description for [Support] tag

Jan DoggenCan we make a better tag wiki for [Support]? The current text is so unclear to me that I cannot even propose a good one. Better Super User through use and working out why things are or how to better your experience here. ?????

 
11:52 AM
0
Q: Recursive convolutional codes

Benjamin LindqvistI'm struggling with the concept of recursive convolutional codes. Say we have the generator matrix $$\begin{pmatrix} \frac{D}{1+D^3} & 1 & 0 \\ \frac{D^2}{1+D^3} & 0 & 1 \end{pmatrix}$$ which means the first column is $[D+D^4+D^7+ \dots; D^2 + D^5 + D^8 + \dots]$. Why doesn't this give the sys...

 
12:14 PM
0
Q: What to do when a particular user is giving low quality answer to his own question and accepting them?

Leo the lionWhile reading Q&A on SO, i found one user who is behaving strange so i checked his profile and found that he is kept asking question and giving the answer of his own asked question, even when they are not really helpful(both question and answers have low quality) and he is accepting as answer. Ev...

 
0
Q: solving an integral1

userHint for solving the following integral: $$\int \frac{2r}{|1-r^{2}|^{\frac{3}{2}}}dr$$? My attempt: I tried putting $u= \frac{1}{\sqrt{|1-r^{2}|}}$ then $du = \frac{r}{(1-r^{2})\sqrt{|1-r^{2}|}}$, but not working. Can someone help me in solving this question? I would be happy to get some hints.

Short title. Title ends with a digit. solving an integral1
0
Q: Miller-Rabin primality test for $2^{32}$+1

mattHow can I prove that $2^{32}$+1 is composite number using Miller-Rabin primality test.I can't find a solution which verify the hypothesis of theorem.Thank you!

 
12:31 PM
-4
Q: Skewing div via CSS

Rehan AslamAttached is a brief mockup of what I need to create. The div not only needs to skew on the bottom, but the next row will need to skew to the top. Is there a clean way this can be done using CSS? I've tried some CSS solutions ( e.g http://jsfiddle.net/mXLgF/ ) but can not get this effect. Each o...

 
0
Q: Estimate parameters of a quadratic function

oopcodeSuppose that we have two data points which tell us about the output of some function f(x): (0, 50) (10, 150) We know that the function is quadratic (so it's something like ax2 + bx + c). The question is: how do we estimate the parameters a, b and c in such a way that the resulting function ma...

Welcome to Math.SE, oopcode. Questions tend to get more attention when they have a tag for a broad area of mathematics relevant to the question. Some of these tags might fit. (autocomment)Normal Human 21 secs ago
0
Q: Does Green's function only work with hermitian operators?

JosephIf I was to use the Green's function method to solve the ordinary differential equation $$Ly=f(x)$$ Would $L$ have to be Hermitian, either way please can you explain why?

Short question. Question contains please. Does Green's function only work with hermitian operators?
0
Q: Closed form expressions for a sum

Dick ArmstrongI was given a sum s, then I expressed it using sigma notation as you can see below. Now, I am supposed to find the closed form values for the said sum as shown in eq.2, but I am stuck at eq.1. I don't know how to deal with the highlighted part, am I forgetting something or did I make some mistake...

Questions tend to get more attention when they have a tag for a broad area of mathematics relevant to the question. Some of these tags might fit. (autocomment)Normal Human 21 secs ago
 
12:54 PM
0
Q: Residue of $\frac{g(z)}{cos^{2}z}$

IDontKnowMathI would like to fin the residue of the function $$\frac{g(z)}{cos^{2}z}$$ at $z_{n}=(n+\frac{1}{2})\pi$, where $g$ is analytic. I tried the Limit formula for higher order poles and it fails. What other method can I use?

0
Q: Exercise 2.20 Pinchover. AN INTRODUCTION TO PARTIAL DIFFERENTIAL EQUATIONS

pomonaConsider the equation $yu_x βˆ’ uu_y = x$. (a) Write a parametric representation of the characteristic curves.

Title contains exercise. Short question. Tagged pde, differential-equations. Exercise 2.20 Pinchover. AN INTRODUCTION TO PARTIAL DIFFERENTIAL EQUATIONS
0
Q: Joint Likelihood

andy. Suppose we have x1,Β·Β·Β· , xn a random sample from a Geometric distribution, say Geom(Ο€x). We also have a second, independent, random sample y1,Β·Β·Β· , ym from Geom(Ο€y). Further we have the following set of hypotheses, H0 : Ο€x = Ο€y against H1 : Ο€x 6= Ο€y. (a) Write down the joint likelihood under ...

Short title. Joint Likelihood
0
Q: Prove ln(n) diverges as quickly as the harmonic series.

Tiamo P.Question: Find a (simple) $f(n)$ so that $\lim\limits_{n \rightarrow \infty} \frac{n \sum\limits_{k=1}^{n} \frac{1}{k}}{f(n)} = 1$ My Attempt: I know the answer, by using Mathematica, is $f(n) = n \cdot ln(n)$. I, however, can't find a way to prove this. I've only tried $$\lim\limits_{n \righta...

 
1:22 PM
0
Q: relationship between fn and f

Lauren BathersSo when looking at functional sequences in terms of uniformally convergent, I am struggling to translate the definitions into examples. I understand that $f_n=\frac{x^2}{n}$ is the sequence $f_n$={$x^2$, $\frac{x^2}{2},$ $\frac{x^2}{3}...$} However how does $f$ relate to $f_n$. If i have a defi...

0
Q: Integral $\frac{1}{\cos^2(z)}$

ChristianHow can I calculate $$\int_{\lvert z-i \rvert = 2} \frac{1}{\cos^2(z)}$$ My first attempt was to use residuals: I have figured out that $z=\pm \pi/2$ is a pole of order $2$, so I tried to apply this formula, but the limit diverges. I have also tried to write $f$ as Laurent-series to get $a_{-1...

0
Q: What is wrong with my calculation for variance?

John TrailI don't get this, I am asked if the following $X_t$ is a brownian motion or not. $Z$ is a standard normal variate. $X_t=\sqrt{t}Z$. I s$X_t$ a Brownian motion? Answer is apparently no and one of the reasons is that the variance of $X_t-X_s=$ is $(\sqrt{t}-\sqrt{s})^2$. But I get $t-s$ as th...

0
Q: Prove that $f$ is not continuous only at the boundary

luka5z It seems obvious, but somehow I find it difficult to prove it in an arbitrary metric space. How can I proceed in this case? Thank you for any help.

Short question. Tagged proof-verification. Prove that $f$ is not continuous only at the boundary
0
Q: Injective immersion is not a flow trajectory

Frank ZermeloLet $M$ be a compact manifold of dimension of $m \geq 2$. Show that there exists an injective immersion of $\mathbb{R}$ in $M$, whose image is not the trajectory of any flow on $M$.

0
Q: Analysis: Proving divergence using partial sums

GeometryHow do I prove that $$\sum_{n=1}^{\infty}\frac{1}{n^s}$$ diverges fos $s<1$, by estimating its partial sums?

0
Q: Integration wrt BM

CSAHow do I integrate: $\int_{\mathbb{R}} (S_t - K)^+ \phi(t) dt$ where $\phi$ is a normal density and $S_t$ is a geometric brownian motion? I know my answer should be $\Phi(d_1)$, where $\Phi$ is the normal CDF and $d_1$ is the $d_1$ appearing in the Black-Scholes price of a European option.

Short title. Integration wrt BM
0
Q: Do the properties of a pedal triangle hold good when the triangle on which the pedal triangle is constructed, an obtuse an triangle?

Rishabh KumarFor Example, Can we assume that the incentre of a pedal triangle is the orthocentre of the triangle on which the pedal triangle is constructed in all cases?

0
Q: Recurrence relation $T(n+1)=T(n)+⌊\sqrt{n+1}βŒ‹$?

Mithlesh UpadhyayConsider the following recurrence relation $T(1)=1$ $T(n+1)=T(n)+⌊\sqrt{n+1}βŒ‹$ for all $nβ‰₯1$ The value of $T(m^2)$ for $mβ‰₯1$ is $(m/6) (21m – 39) + 4$ $(m/6) (4m^2 – 3m + 5)$ $(m/2) (m^{2.5} – 11m + 20) – 5$ $(m/6) (5m^3 – 34m^2 + 137m – 104) + (5/6)$ My attempt : I've used counter exam...

0
Q: How can i proof that the follow polynomial is irriducible in $\mathbb{Q}$?

SkillsHow can i proof that $x^5 + 7x^4 + 2x^3 + 6x^2 - x + 8$ is irrudicible in $\mathbb{Q}$? I can't use the Eisenstein's criterion and I tryed to put this polynomial in $\mathbb{Z}_3$ and $\mathbb{Z}_5$. Can you give me some advice please?

0
Q: Jordan Canonical form question

turtleSuppose $A$ is a 4 by 4 matrix with the characteristic polynomial $P_A(\lambda) = (\lambda-2)^4$ with the minimal polynomial $m_A(\lambda) = (\lambda -2)^2$. This tells me that that the Jordan form with respect to the eigenvalue 2 (the only eigenvalue) is $4 \times 4$ size matrix and the largest ...

Words such as question are uninformative in titles. Please edit the title so that it better describes the specifics of your question. Do not hesitate to make it longer or include a formula if needed. More tips here. (autocomment)Normal Human 21 secs ago
0
Q: If a function in uniformly continuous then its bounded

Maths beginnerSuppose that function f:[0,1)->R is uniformly continuous on [0,1). Prove that the function f is bounded (i.e. that range(f) is a bounded set)

0
Q: $f \in \mathbb Z[x]$ be irreducible and suppose $f(x)$ has two roots in $\mathbb C$ with product $1$ , then degree of $f$ is even ?

Saun DevLet $f \in \mathbb Z[x]$ be irreducible and suppose $f(x)$ has two roots in $\mathbb C$ with product $1$ . Then is it true that degree of $f$ is even ?

Short question. [$f \in \mathbb Z[x]$ be irreducible and suppose $f(x)$ has two roots in $\mathbb C$ with product $1$ , then degree of $f$ is even ?](math.stackexchange.com/q/1564176)
0
Q: Maximum of martingales

OscarI need to show whether or not the maximum of two martingales is also a martingale. Originally, I thought yes. But supposedly the answer is no. So as a counter-example, let $U_i$ be $iid$ $unif(0,1)$, $X_0 = 1$, and $$X_n = 2^n\prod_{k=1}^n U_k.\tag{1}$$ I already know that $(X_n)$ is a martingal...

0
Q: Combinatorics How Many Tree

Serkan KlvzHow can i prove for any tree G=(V,E) |E|=|V|-1 I have tried the induction on the number of vertices but nothing happened

Short title. Short question. Combinatorics How Many Tree
0
Q: the limits of integral for the joint probability density function

A.D The joint probability density function of X and Y is given by f(x,y)=c*(y^2 - 36*^2)*e^{-y} y/6 < x < y/6 0 < y < infty Find c and the expected value of X: That's my question that I have an issue. I know how can I find c and expected value of x or y for joint pdf. When...

This site uses MathJax formatting of formulas. More tips here. (from a bot)Normal Human 21 secs ago
0
Q: Inverse of $(I+xy^*)$

XentiusHere is the question: What is the simplest expression for the inverse of $(I+xy^*)$ where $x$ an $y$ are $n*1$ vectors, also state the required condition for the invertibility. I tried to solve it by using determinant, however it did not lead to anywhere. Could you please give me a hand?

Short title. Question contains please. Inverse of $(I+xy^*)$
 
2:26 PM
0
Q: Why was this deleted and not merged/marked duplicate? (Discussion)

Ashhar HasanI know my answers on these two (1 and 2) were exact duplicates and are not encouraged and should be deleted. So no wrong on that part. But since both the questions are also possible duplicates and have been flagged as such, why delete those instead of merge them? Also, how can I improve this on...

 
0
Q: Finding extremes for function $f(t,x,y,z)

MiladFind the extremal values of $f(t,x,y,z)=t^2+3x^2+0.3y^2+12z^2 on the unit sphere in $R^4. Justify your answer.

0
Q: Uniform convergence

darrenDoes this series converge uniformly or not ? sum from n=1 to infinity of (nx-1)/n^2 in the interval of [2,5] I know i can use the the Weierstrass M-test can be use to show if a series converges uniformly but how would i show that my series does not converge uniformly ?

Short title. Uniform convergence
0
Q: how to prove this function is continuous in (-1,1)

daydayupHow to prove $f(x)=\frac{x}{(1-x)^2}$ is continuous in (-1,1) using either $\epsilon-\delta $ definition or other methods. thank u

0
Q: Finding the sum of n terms $S_n$ starting from sigma $k=0$

Mathy$\sum_{k=0}^{n} ((4n-3)*2^n)+4$ = $(2^{n+3}+4)n-7*2^{n+1}+15$ How? I've tried everything but i don't see it. Any equivalent solutions are also welcome, thanks.

0
Q: COMPLEX NUMBER AND DEFΔ°NE

fsuluovaDo that question have a definity and Δ±f that question have a definity set, what is the definity set for that question. $0^i$

Short title. Short question. COMPLEX NUMBER AND DEFΔ°NE
0
Q: Elementary Analysis Continuity

YasLet f : R-->R be continuous at p in R: If f(p) > 0; then show that there is an open interval I that contains p such that f(x) > 0 for all x in I:

Welcome to Math.SE, Yas. Consider replacing (analysis) with a more specific tag for the relevant branch of analysis. (autocomment)Normal Human 21 secs ago
0
Q: Need Solution (Please Help)

IT Solution Given the matrix representing a relation on a finite set, write a program in C to determine whether the relation is reflexive and/or irreflexive. Given the matrix representing a relation on a finite set, write a program in C to determine whether the relation is symmetric and/or antisymmetric. Gi...

Welcome to Math.SE, IT Solution. Words such as please, help are uninformative in titles. Please edit the title so that it better describes the specifics of your question. Do not hesitate to make it longer or include a formula if needed. More tips here. (autocomment)Normal Human 21 secs ago
0
Q: Show that for this rotation matrix <Ax, Ay> = <x,y> using rotation matrix and any vectors

BrendanShow that for the rotation matrix $begin{amatrix}cos(a) & -sin(a)\\sin(a) & cos(a)\end{amatrix}$ = With x and y being any vectors contained in R^2

0
Q: Hasse diagram (representation)

user284901Is it possible to draw more then one Hasse diagram for one/same poset ? $ R = \{(1,1); (2,2); (3,3); (4,4); (1,3); (1,4); (2,4); (3,4)\} $ There is no the smallest element, the greatest element is 4. Minimal elements are 1, 2. Maximal element is 4. I drew two Hasse diagrams for this relation...

 
2:46 PM
0
Q: Which in case of duplicate questions should prevail?

mathreadlerIs there any routine to decide which one of the duplicate questions should prevail? The one which has been in existance the longest? It would probably not be good if both were accidentally shut down because they were each others duplicate.

-1
Q: Option to add questions to reading list/subscribe to them?

Ashhar HasanI have some questions I feel like saying "Me Too!!!" to and some that are interesting enough so that I would like to keep myself posted on developments like this and this. Would be awesome if I didn't have to try writing a Python script to notify me of changes on the page OR having to bookmark t...

 
0
Q: Prove for any vector

piternetI've got to prove for any vector given, $\vec{x} = [x_{1}, ..., x_{n}]^{T} \in \mathbb{C}^{n}$ that it's true that: $\left | x_{1} + ... + x_{n} \right | \leqslant \sqrt{n} \cdot \left \| x \right \| $ Please give me some hints for that. I know that in this case: $\left \| x \right \| = \sqrt{\le...

Short title. Question contains please. Prove for any vector
0
Q: Cantor set and triadic expansions

Kasper CoolsI'm trying to prove that the Cantor set is equal to a certain set of 'escape points' for a mathematical feedback system. In this proof I'm going to need the fact that every element of the Cantor set has a base-3 representation in which only 0's and 2's occur. However, I'm having a hard time with...

Consider adding a tag for a broader subject area to which the question belongs. Some of these tags might fit. (from a bot)Normal Human 21 secs ago
0
Q: Show <,> is not an inner product

JasimudI'm trying to figure out a problem on inner space products: Les S be the space of $C^1[-1,1]$ functions, and: <;> $\mapsto \int_{-1}^{1} f'(x)g'(x)dx$ Decide if <;> is an inner product over S. To decide if this is an inner product I'm going through the axioms, and I can't show this one...

Tagged proof-verification. Show <,> is not an inner product
0
Q: Is $A$ a convex set?

mahdi moosazadehDraw a Cantor set $C$ on the circle and consider the set $A$ of all chords between points of $C$. Is $A$ a convex set?

Short title. Short question. Is $A$ a convex set?
0
Q: An automorphism of order $2$

ghazaleLet $G$ be a connected bipartite graph, so $GΓ—G$ is bipartite and has exactly two components. Show that at least one component of $GΓ—G$ admits an involution (i.e., an automorphism of order $2$) that interchanges its partite sets. An automorphism of order $2$ is simply an automorphism $Ο•β‰ idΟ•$ s...

 
00:00 - 15:0015:00 - 00:00

« first day (35 days earlier)      last day (539 days later) »